When to use the root test. Is this not a good situation to use it? The 2019 Stack Overflow...

How technical should a Scrum Master be to effectively remove impediments?

For what reasons would an animal species NOT cross a *horizontal* land bridge?

Why isn't the circumferential light around the M87 black hole's event horizon symmetric?

Deal with toxic manager when you can't quit

Can a flute soloist sit?

Time travel alters history but people keep saying nothing's changed

What is the most effective way of iterating a std::vector and why?

How to save as into a customized destination on macOS?

If I score a critical hit on an 18 or higher, what are my chances of getting a critical hit if I roll 3d20?

When should I buy a clipper card after flying to OAK?

slides for 30min~1hr skype tenure track application interview

What do the Banks children have against barley water?

What is the motivation for a law requiring 2 parties to consent for recording a conversation

Does coating your armor in silver add any effects?

Does a dangling wire really electrocute me if I'm standing in water?

Where to refill my bottle in India?

Multiply Two Integer Polynomials

Can we generate random numbers using irrational numbers like π and e?

Is a "Democratic" Oligarchy-Style System Possible?

Falsification in Math vs Science

Is there a symbol for a right arrow with a square in the middle?

Why didn't the Event Horizon Telescope team mention Sagittarius A*?

Origin of "cooter" meaning "vagina"

Looking for Correct Greek Translation for Heraclitus



When to use the root test. Is this not a good situation to use it?



The 2019 Stack Overflow Developer Survey Results Are InWhich test would be appropriate to use on this series to show convergence/divergence?Integral test vs root test vs ratio testHow to show convergence or divergence of a series when the ratio test is inconclusive?Root test with nested power function?Confused about using alternating test, ratio test, and root test (please help).Radius and interval of convergence of $sum_{n=1}^{infty}(-1)^nfrac{x^{2n}}{(2n)!}$ by root and ratio test are different?How would I use root/ratio test on $sum_{n=1}^inftyleft(frac{n}{n+1}right)^{n^2}$?How would I know when to use what test for convergence?convergence of a sum fails with root testIntuition for Root Test.












2












$begingroup$


I'm having trouble seeing when to use the root test. nth powers occur, but I think the ratio test is easier:



enter image description here



Here is the problem:



$$sum_{n=1}^{infty} frac{x^n}{n^44^n}$$



So the ratio test seems to work here, but can't the root test be used to? The problem is that the $n^4$ doesnt play well with the root test right?



Here is the beginning of my solution with the ratio test:



$$biggr lbrack frac{a_{n+1}}{a_n} biggr rbrack = biggr lbrack frac{x^{n+1}}{(n+1)^4 * 4^{n+1}} * frac{n^4*4^n}{x^n} biggr rbrack = biggr lbrack frac{x*n^4}{(n+1)^4 * 4} biggr rbrack = frac{x}{4}$$



So I don't think the explanation for when to use the root test is totally right right? I can't really use it here because the $n^4$ causes some problems with the root test right?










share|cite|improve this question









$endgroup$

















    2












    $begingroup$


    I'm having trouble seeing when to use the root test. nth powers occur, but I think the ratio test is easier:



    enter image description here



    Here is the problem:



    $$sum_{n=1}^{infty} frac{x^n}{n^44^n}$$



    So the ratio test seems to work here, but can't the root test be used to? The problem is that the $n^4$ doesnt play well with the root test right?



    Here is the beginning of my solution with the ratio test:



    $$biggr lbrack frac{a_{n+1}}{a_n} biggr rbrack = biggr lbrack frac{x^{n+1}}{(n+1)^4 * 4^{n+1}} * frac{n^4*4^n}{x^n} biggr rbrack = biggr lbrack frac{x*n^4}{(n+1)^4 * 4} biggr rbrack = frac{x}{4}$$



    So I don't think the explanation for when to use the root test is totally right right? I can't really use it here because the $n^4$ causes some problems with the root test right?










    share|cite|improve this question









    $endgroup$















      2












      2








      2





      $begingroup$


      I'm having trouble seeing when to use the root test. nth powers occur, but I think the ratio test is easier:



      enter image description here



      Here is the problem:



      $$sum_{n=1}^{infty} frac{x^n}{n^44^n}$$



      So the ratio test seems to work here, but can't the root test be used to? The problem is that the $n^4$ doesnt play well with the root test right?



      Here is the beginning of my solution with the ratio test:



      $$biggr lbrack frac{a_{n+1}}{a_n} biggr rbrack = biggr lbrack frac{x^{n+1}}{(n+1)^4 * 4^{n+1}} * frac{n^4*4^n}{x^n} biggr rbrack = biggr lbrack frac{x*n^4}{(n+1)^4 * 4} biggr rbrack = frac{x}{4}$$



      So I don't think the explanation for when to use the root test is totally right right? I can't really use it here because the $n^4$ causes some problems with the root test right?










      share|cite|improve this question









      $endgroup$




      I'm having trouble seeing when to use the root test. nth powers occur, but I think the ratio test is easier:



      enter image description here



      Here is the problem:



      $$sum_{n=1}^{infty} frac{x^n}{n^44^n}$$



      So the ratio test seems to work here, but can't the root test be used to? The problem is that the $n^4$ doesnt play well with the root test right?



      Here is the beginning of my solution with the ratio test:



      $$biggr lbrack frac{a_{n+1}}{a_n} biggr rbrack = biggr lbrack frac{x^{n+1}}{(n+1)^4 * 4^{n+1}} * frac{n^4*4^n}{x^n} biggr rbrack = biggr lbrack frac{x*n^4}{(n+1)^4 * 4} biggr rbrack = frac{x}{4}$$



      So I don't think the explanation for when to use the root test is totally right right? I can't really use it here because the $n^4$ causes some problems with the root test right?







      sequences-and-series






      share|cite|improve this question













      share|cite|improve this question











      share|cite|improve this question




      share|cite|improve this question










      asked yesterday









      Jwan622Jwan622

      2,38711632




      2,38711632






















          2 Answers
          2






          active

          oldest

          votes


















          3












          $begingroup$

          When doing a root test,
          powers of $n$ can be ignored
          because,
          for any fixed $k$,



          $lim_{n to infty} (n^k)^{1/n}
          =1
          $
          .



          This is because
          $ (n^k)^{1/n}
          =n^{k/n}
          =e^{k ln(n)/n}
          $

          and
          $lim_{n to infty} frac{ln(n)}{n}
          =0$
          .



          An easy,
          but nonelementary proof of this is this:



          $begin{array}\
          ln(n)
          &=int_1^n dfrac{dt}{t}\
          &<int_1^n dfrac{dt}{t^{1/2}}\
          &=2t^{1/2}|_1^n\
          &lt 2sqrt{n}\
          text{so}\
          dfrac{ln(n)}{n}
          &<dfrac{2}{sqrt{n}}\
          end{array}
          $



          Therefore
          $ (n^k)^{1/n}
          =n^{k/n}
          =e^{k ln(n)/n}
          lt e^{2k/sqrt{n}}
          to 1
          $
          .






          share|cite|improve this answer









          $endgroup$





















            4












            $begingroup$

            It doesn't cause any problems, because $lim_{ntoinfty}sqrt[n]{n^4}=1.$ Actually, the root test is stronger than the ratio test. Sometimes the root test limit exists, but the ratio test limit does not. However, if they both exist, then they are equal. Which is why if one limit is $1$ you shouldn't try the other, even though the root test is stronger.






            share|cite|improve this answer









            $endgroup$














              Your Answer





              StackExchange.ifUsing("editor", function () {
              return StackExchange.using("mathjaxEditing", function () {
              StackExchange.MarkdownEditor.creationCallbacks.add(function (editor, postfix) {
              StackExchange.mathjaxEditing.prepareWmdForMathJax(editor, postfix, [["$", "$"], ["\\(","\\)"]]);
              });
              });
              }, "mathjax-editing");

              StackExchange.ready(function() {
              var channelOptions = {
              tags: "".split(" "),
              id: "69"
              };
              initTagRenderer("".split(" "), "".split(" "), channelOptions);

              StackExchange.using("externalEditor", function() {
              // Have to fire editor after snippets, if snippets enabled
              if (StackExchange.settings.snippets.snippetsEnabled) {
              StackExchange.using("snippets", function() {
              createEditor();
              });
              }
              else {
              createEditor();
              }
              });

              function createEditor() {
              StackExchange.prepareEditor({
              heartbeatType: 'answer',
              autoActivateHeartbeat: false,
              convertImagesToLinks: true,
              noModals: true,
              showLowRepImageUploadWarning: true,
              reputationToPostImages: 10,
              bindNavPrevention: true,
              postfix: "",
              imageUploader: {
              brandingHtml: "Powered by u003ca class="icon-imgur-white" href="https://imgur.com/"u003eu003c/au003e",
              contentPolicyHtml: "User contributions licensed under u003ca href="https://creativecommons.org/licenses/by-sa/3.0/"u003ecc by-sa 3.0 with attribution requiredu003c/au003e u003ca href="https://stackoverflow.com/legal/content-policy"u003e(content policy)u003c/au003e",
              allowUrls: true
              },
              noCode: true, onDemand: true,
              discardSelector: ".discard-answer"
              ,immediatelyShowMarkdownHelp:true
              });


              }
              });














              draft saved

              draft discarded


















              StackExchange.ready(
              function () {
              StackExchange.openid.initPostLogin('.new-post-login', 'https%3a%2f%2fmath.stackexchange.com%2fquestions%2f3181802%2fwhen-to-use-the-root-test-is-this-not-a-good-situation-to-use-it%23new-answer', 'question_page');
              }
              );

              Post as a guest















              Required, but never shown

























              2 Answers
              2






              active

              oldest

              votes








              2 Answers
              2






              active

              oldest

              votes









              active

              oldest

              votes






              active

              oldest

              votes









              3












              $begingroup$

              When doing a root test,
              powers of $n$ can be ignored
              because,
              for any fixed $k$,



              $lim_{n to infty} (n^k)^{1/n}
              =1
              $
              .



              This is because
              $ (n^k)^{1/n}
              =n^{k/n}
              =e^{k ln(n)/n}
              $

              and
              $lim_{n to infty} frac{ln(n)}{n}
              =0$
              .



              An easy,
              but nonelementary proof of this is this:



              $begin{array}\
              ln(n)
              &=int_1^n dfrac{dt}{t}\
              &<int_1^n dfrac{dt}{t^{1/2}}\
              &=2t^{1/2}|_1^n\
              &lt 2sqrt{n}\
              text{so}\
              dfrac{ln(n)}{n}
              &<dfrac{2}{sqrt{n}}\
              end{array}
              $



              Therefore
              $ (n^k)^{1/n}
              =n^{k/n}
              =e^{k ln(n)/n}
              lt e^{2k/sqrt{n}}
              to 1
              $
              .






              share|cite|improve this answer









              $endgroup$


















                3












                $begingroup$

                When doing a root test,
                powers of $n$ can be ignored
                because,
                for any fixed $k$,



                $lim_{n to infty} (n^k)^{1/n}
                =1
                $
                .



                This is because
                $ (n^k)^{1/n}
                =n^{k/n}
                =e^{k ln(n)/n}
                $

                and
                $lim_{n to infty} frac{ln(n)}{n}
                =0$
                .



                An easy,
                but nonelementary proof of this is this:



                $begin{array}\
                ln(n)
                &=int_1^n dfrac{dt}{t}\
                &<int_1^n dfrac{dt}{t^{1/2}}\
                &=2t^{1/2}|_1^n\
                &lt 2sqrt{n}\
                text{so}\
                dfrac{ln(n)}{n}
                &<dfrac{2}{sqrt{n}}\
                end{array}
                $



                Therefore
                $ (n^k)^{1/n}
                =n^{k/n}
                =e^{k ln(n)/n}
                lt e^{2k/sqrt{n}}
                to 1
                $
                .






                share|cite|improve this answer









                $endgroup$
















                  3












                  3








                  3





                  $begingroup$

                  When doing a root test,
                  powers of $n$ can be ignored
                  because,
                  for any fixed $k$,



                  $lim_{n to infty} (n^k)^{1/n}
                  =1
                  $
                  .



                  This is because
                  $ (n^k)^{1/n}
                  =n^{k/n}
                  =e^{k ln(n)/n}
                  $

                  and
                  $lim_{n to infty} frac{ln(n)}{n}
                  =0$
                  .



                  An easy,
                  but nonelementary proof of this is this:



                  $begin{array}\
                  ln(n)
                  &=int_1^n dfrac{dt}{t}\
                  &<int_1^n dfrac{dt}{t^{1/2}}\
                  &=2t^{1/2}|_1^n\
                  &lt 2sqrt{n}\
                  text{so}\
                  dfrac{ln(n)}{n}
                  &<dfrac{2}{sqrt{n}}\
                  end{array}
                  $



                  Therefore
                  $ (n^k)^{1/n}
                  =n^{k/n}
                  =e^{k ln(n)/n}
                  lt e^{2k/sqrt{n}}
                  to 1
                  $
                  .






                  share|cite|improve this answer









                  $endgroup$



                  When doing a root test,
                  powers of $n$ can be ignored
                  because,
                  for any fixed $k$,



                  $lim_{n to infty} (n^k)^{1/n}
                  =1
                  $
                  .



                  This is because
                  $ (n^k)^{1/n}
                  =n^{k/n}
                  =e^{k ln(n)/n}
                  $

                  and
                  $lim_{n to infty} frac{ln(n)}{n}
                  =0$
                  .



                  An easy,
                  but nonelementary proof of this is this:



                  $begin{array}\
                  ln(n)
                  &=int_1^n dfrac{dt}{t}\
                  &<int_1^n dfrac{dt}{t^{1/2}}\
                  &=2t^{1/2}|_1^n\
                  &lt 2sqrt{n}\
                  text{so}\
                  dfrac{ln(n)}{n}
                  &<dfrac{2}{sqrt{n}}\
                  end{array}
                  $



                  Therefore
                  $ (n^k)^{1/n}
                  =n^{k/n}
                  =e^{k ln(n)/n}
                  lt e^{2k/sqrt{n}}
                  to 1
                  $
                  .







                  share|cite|improve this answer












                  share|cite|improve this answer



                  share|cite|improve this answer










                  answered yesterday









                  marty cohenmarty cohen

                  75.2k549130




                  75.2k549130























                      4












                      $begingroup$

                      It doesn't cause any problems, because $lim_{ntoinfty}sqrt[n]{n^4}=1.$ Actually, the root test is stronger than the ratio test. Sometimes the root test limit exists, but the ratio test limit does not. However, if they both exist, then they are equal. Which is why if one limit is $1$ you shouldn't try the other, even though the root test is stronger.






                      share|cite|improve this answer









                      $endgroup$


















                        4












                        $begingroup$

                        It doesn't cause any problems, because $lim_{ntoinfty}sqrt[n]{n^4}=1.$ Actually, the root test is stronger than the ratio test. Sometimes the root test limit exists, but the ratio test limit does not. However, if they both exist, then they are equal. Which is why if one limit is $1$ you shouldn't try the other, even though the root test is stronger.






                        share|cite|improve this answer









                        $endgroup$
















                          4












                          4








                          4





                          $begingroup$

                          It doesn't cause any problems, because $lim_{ntoinfty}sqrt[n]{n^4}=1.$ Actually, the root test is stronger than the ratio test. Sometimes the root test limit exists, but the ratio test limit does not. However, if they both exist, then they are equal. Which is why if one limit is $1$ you shouldn't try the other, even though the root test is stronger.






                          share|cite|improve this answer









                          $endgroup$



                          It doesn't cause any problems, because $lim_{ntoinfty}sqrt[n]{n^4}=1.$ Actually, the root test is stronger than the ratio test. Sometimes the root test limit exists, but the ratio test limit does not. However, if they both exist, then they are equal. Which is why if one limit is $1$ you shouldn't try the other, even though the root test is stronger.







                          share|cite|improve this answer












                          share|cite|improve this answer



                          share|cite|improve this answer










                          answered yesterday









                          MelodyMelody

                          1,09412




                          1,09412






























                              draft saved

                              draft discarded




















































                              Thanks for contributing an answer to Mathematics Stack Exchange!


                              • Please be sure to answer the question. Provide details and share your research!

                              But avoid



                              • Asking for help, clarification, or responding to other answers.

                              • Making statements based on opinion; back them up with references or personal experience.


                              Use MathJax to format equations. MathJax reference.


                              To learn more, see our tips on writing great answers.




                              draft saved


                              draft discarded














                              StackExchange.ready(
                              function () {
                              StackExchange.openid.initPostLogin('.new-post-login', 'https%3a%2f%2fmath.stackexchange.com%2fquestions%2f3181802%2fwhen-to-use-the-root-test-is-this-not-a-good-situation-to-use-it%23new-answer', 'question_page');
                              }
                              );

                              Post as a guest















                              Required, but never shown





















































                              Required, but never shown














                              Required, but never shown












                              Required, but never shown







                              Required, but never shown

































                              Required, but never shown














                              Required, but never shown












                              Required, but never shown







                              Required, but never shown







                              Popular posts from this blog

                              Taj Mahal Inhaltsverzeichnis Aufbau | Geschichte | 350-Jahr-Feier | Heutige Bedeutung | Siehe auch |...

                              Baia Sprie Cuprins Etimologie | Istorie | Demografie | Politică și administrație | Arii naturale...

                              Ciclooctatetraenă Vezi și | Bibliografie | Meniu de navigare637866text4148569-500570979m